Question about absolute value limits?

Click For Summary
When evaluating limits involving absolute values, it is crucial to check both the left and right limits when the expression inside the absolute value equals zero at the limit point. For example, in the limit as x approaches 3/2 of (2x^2-3x)/|2x-3|, the behavior of the absolute value changes depending on whether x is slightly greater or less than 3/2. Conversely, in the limit as x approaches -2 of (2-|x|)/(2+x), the absolute value does not affect the expression differently for values just above or below -2, leading to the same result. This distinction is essential for correctly determining the limit's value. Understanding these nuances is key to solving absolute value limits effectively.
emlekarc
Messages
27
Reaction score
0
When do you check the limit from the right and left of a limit with an absolute value in the numerator or denominator?

For example, why do you check the limit from both sides of:

Lim x -> 3/2 (2x^2-3x)/absolute value(2x-3)

But only the left side of:

limit as x approaches -2

(2-absolute value x)/(2+x)
 
Physics news on Phys.org
If the thing inside the absolute value is equal to zero when you plug in the limiting value of x, then depending on whether x is a little bigger or a little smaller it will change how the absolute value acts on the expression inside of it. For example

\lim_{x\to 3/2} \frac{...}{|2x-3|}
if x = 3/2, 2x-3 = 0. So if x > 3/2, |2x-3| = 2x-3, but if x < 3/2, |2x-3| = 3-2x and you need to be careful about this distinction.

On the other hand, if x = -2, 2-x = 4. So if x > -2 by a little bit, |2-x| = 2-x, but if x < -2 by a little bit, |2-x| = 2-x still. In both cases you get the same expression when you drop the absolute value sign.
 
Question: A clock's minute hand has length 4 and its hour hand has length 3. What is the distance between the tips at the moment when it is increasing most rapidly?(Putnam Exam Question) Answer: Making assumption that both the hands moves at constant angular velocities, the answer is ## \sqrt{7} .## But don't you think this assumption is somewhat doubtful and wrong?

Similar threads

  • · Replies 12 ·
Replies
12
Views
3K
Replies
3
Views
1K
  • · Replies 8 ·
Replies
8
Views
2K
  • · Replies 2 ·
Replies
2
Views
1K
  • · Replies 25 ·
Replies
25
Views
2K
  • · Replies 5 ·
Replies
5
Views
2K
  • · Replies 1 ·
Replies
1
Views
2K
  • · Replies 8 ·
Replies
8
Views
2K
  • · Replies 2 ·
Replies
2
Views
1K
  • · Replies 2 ·
Replies
2
Views
1K